Tài liệu chuyên toán - Bất đẳng thức hiện đại - phần 9 pdf

30 457 1
Tài liệu chuyên toán - Bất đẳng thức hiện đại - phần 9 pdf

Đang tải... (xem toàn văn)

Tài liệu hạn chế xem trước, để xem đầy đủ mời bạn chọn Tải xuống

Thông tin tài liệu

233 x + 2y = a b + 6b a + 4b c + 3a c + 4c a + a 2 bc + 2b 2 ca  12  a b + 6b a + 4b c + 3a c + 4c a + a 2 b  a + 2b 2 b  a  12 = 2a b + 8b a + 4b c + 3a c + 4c a  12 = 2  a b + 4b a  + 4b c + 3  a c + c a  + c a  12  8 + 4 + 6 + 0 12 = 6 > 0 Do đó Nếu y  0 thì ta có X cy c z(a  b) 2  y(a  c) 2 + z(a  b) 2  (y + z)(a  b) 2  0 Nếu y  0 thì ta có X cy c z(a  b) 2  x(b  c) 2 + y[2(b  c) 2 + 2(a  b) 2 ] + z(a  b) 2 = (x + 2y)(b  c) 2 + (z + 2y)(a  b) 2  0: Bổ đề được chứng minh xong. Trở lại bài toán của ta, đặt a = 1 x ; b = 1 y ; c = 1 z thì bất đẳng thức trở thành X cy c a b  3 X cy c bc 2a 2 + bc , X cy c a b + 6 X cy c a 2 2a 2 + bc  9 Sử dụng bất đẳng thức Cauchy Schwarz, ta có X cy c a 2 2a 2 + bc  P cy c a ! 2 2 P cy c a 2 + P cy c bc Mặt khác, theo bổ đề trên, ta có X cy c a b  9 P cy c a 2 P cy c a ! 2 234 CHƯƠNG 2. SÁNG TẠO BẤT ĐẲNG THỨC Nên ta chỉ cần chứng minh được 3 P cy c a 2 P cy c a ! 2 + 2 P cy c a ! 2 2 P cy c a 2 + P cy c bc  3 Đặt t = P cyc bc P cyc a ! 2  1 3 thì bất đẳng thức trở thành 3(1  2t) + 2 2(1  2t) + t  3 , 2(1  3t) 2 2  3t  0: Bất đẳng thức cuối hiển nhiên đúng nên ta có đpcm. Đẳng thức xảy ra khi và chỉ khi x = y = z: Bài toán 2.30 Cho các số không âm a; b; c; không có 2 số nào đồng thời bằng 0: Chứng minh rằng a 4 a 3 + b 3 + b 4 b 3 + c 3 + c 4 c 3 + a 3  a + b + c 2 : (Vasile Cirtoaje) Lời giải. Sử dụng bất đẳng thức Cauchy Schwarz, ta có X cy c a 4 a 3 + b 3 !" X cy c a 2 (a 3 + b 3 ) #  X cy c a 3 ! 2 Ta cần chứng minh 2 X cy c a 3 ! 2  X cy c a !" X cy c a 2 (a 3 + b 3 ) # 235 Sử dụng bất đẳng thức Vasile P cy c ab 3  1 3 P cy c a 2 ! 2 , ta có X cy c a 2 (a 3 + b 3 ) = X cy c a 5 + P cy c ab P cy c a X cy c ab 3 + X cy c a 2 b 2 !  abc X cy c a 2 !  X cy c a 5 + P cy c ab P cy c a 2 4 1 3 X cy c a 2 ! 2 + X cy c a 2 b 2 3 5  abc X cy c a 2 ! Do tính thuần nhất, ta có thể chuẩn hóa cho a+b+c = 1. Đặt q = ab+bc+ca; r = abc, khi đó ta có X cy c a 5 + P cy c ab P cy c a 2 4 1 3 X cy c a 2 ! 2 + X cy c a 2 b 2 3 5  abc X cy c a 2 ! = 1 3 [3(4  5q)r + 3  14q + 11q 2 + 7q 3 ] và X cy c a 3 ! 2 = (1 3q + 3r) 2 Ta phải chứng minh 2(1  3q + 3r) 2  1 3 [3(4  5q)r + 3  14q + 11q 2 + 7q 3 ] , 54r 2 + 3(8  28q)r + 3  22q + 43q 2  7q 3  9qr  0 Sử dụng bất đẳng thức Schur bậc 3, ta có r  4q 1 9 và chú ý rằng f(r) = 54r 2 + 3(8 28q)r tăng với mọi r  4q 1 9 , ta có f(r) + 3  22q + 43q 2  7q 3  9qr  f  4q  1 9  + 3  22q + 43q 2  7q 3  9qr = 1  22 3 q + 49 3 q 2  7q 3  9qr Từ bất đẳng thức (a  b) 2 (b  c) 2 (c  a) 2  0, ta suy ra được r  1 27 h 9q  2 + 2(1  3q) p 1  3q i 236 CHƯƠNG 2. SÁNG TẠO BẤT ĐẲNG THỨC Do đó, ta chỉ cần chứng minh 1  22 3 q + 49 3 q 2  7q 3  1 3 q h 9q  2 + 2(1  3q) p 1  3q i  0 , 1 3 (1  3q)  7q 2  11q + 3  2q p 1  3q   0 Sử dụng bất đẳng thức AM-GM, ta có 7q 2  11q + 3  2q p 1  3q  7q 2  11q + 3  (q 2 + 1  3q) = 2(1  q)(1  3q)  0: Bất đẳng thức được chứng minh. Đẳng thức xảy ra khi và chỉ khi a = b = c: Bài toán 2.31 Cho các số dương a; b; c: Chứng minh rằng 3 p (a + b) 2 (b + c) 2 (c + a) 2  3 p (a  b) 2 (b  c) 2 (c  a) 2 + 4 3 p a 2 b 2 c 2 : (Sung Yoon Kim) Lời giải. Sử dụng bất đẳng thức Cauchy Schwarz, ta có 3 s (a  b) 2 (b  c) 2 (c  a) 2 (a + b) 2 (b + c) 2 (a + c) 2 + 4 3 s a 2 b 2 c 2 (a + b) 2 (b + c) 2 (a + c) 2 = 3 s (a  b) 2 (a + b) 2  (b  c) 2 (b + c) 2  (c  a) 2 (c + a) 2 + 4 3 s ab (a + b) 2  bc (b + c) 2  ca (c + a) 2  1 3 X cy c (a  b) 2 (a + b) 2 + 4 3 X cy c ab (a + b) 2 = 1 3 X cy c  (a  b) 2 (a + b) 2 + 4ab (a + b) 2  = 1: Đẳng thức xảy ra khi và chỉ khi a = b = c: Bài toán 2.32 Cho các số dương a; b; c thỏa mãn abc = 1: Tìm hằng số k lớn nhất sao cho bất đẳng thức sau đúng 1 a(1 + bc) 2 + 1 b(1 + ca) 2 + 1 c(1 + ab) 2  k (1 + ab)(1 + bc)(1 + ca) + 3 4  k 8 (Võ Quốc Bá Cẩn) Lời giải 1. Cho a = 2; b = 1; c = 1 2 ; ta được k  4: Ta sẽ chứng minh đây là giá trị mà ta cần tìm, tức là 4 X cy c 1 a(1 + bc) 2  1 + 16 (1 + ab)(1 + bc)(1 + ca) 237 , 4 X cy c a (a + 1) 2  1 + 16 (1 + a)(1 + b)(1 + c) Đặt x = 1a 1+a ; y = 1b 1+b ; z = 1c 1+c ; thì ta có x; y; z 2 [1; 1] và (1  x)(1  y)(1  z) = (1 + x)(1 + y)(1 + z) ) x + y + z + xyz = 0 Bất đẳng thức trở thành X cy c (1  x 2 )  1 + 2(1 + x)(1 + y)(1 + z) , x 2 + y 2 + z 2 + 2(xy + yz + zx) + 2(x + y + z + xyz)  0 , (x + y + z) 2  0 hiển nhiên đúng. Bất đẳng thức được chứng minh. Vậy ta đi đến kết luận k max = 4: Lời giải 2. Tương tự như trên, ta cần phải chứng minh 4 X cy c 1 a(1 + bc) 2  1 + 16 (1 + ab)(1 + bc)(1 + ca) Vì a; b; c > 0; abc = 1 nên tồn tại các số x; y ; z > 0 sao cho a = x y ; b = y z ; c = z x . Bất đẳng thức trở thành X cy c xy (x + y) 2  1 4 + 4xyz (x + y)(y + z)(z + x) 238 CHƯƠNG 2. SÁNG TẠO BẤT ĐẲNG THỨC Ta có 4xyz (x + y)(y + z)(z + x) = 2[(x + y)(y + z)(z + x)  xy(x + y)  yz(y + z)  zx(z + x)] (x + y)(y + z)(z + x) = 2  P cy c (x 2 + yz)(y + z) (x + y)(y + z)(z + x) = 2  X cy c x x + y  x x + z + X cy c y x + y  z x + z ! = 2  " X cy c x x + y ! X cy c y x + z !  X cy c x x + y  y x + y # = 2  X cy c x x + y ! X cy c y x + z ! + X cy c xy (x + y) 2 Bất đẳng thức tương đương với X cy c x x + y ! X cy c y x + z !  9 4 Sử dụng bất đẳng thức AM-GM, ta có X cy c x x + y ! X cy c y x + z !  1 4 X cy c x x + y + X cy c y x + y ! 2 = 9 4 : Bài toán 2.33 Cho các số không âm a; b; c thỏa mãn a 2 + b 2 + c 2 = 1: Tìm giá trị lớn nhất của biểu thức P = (a b)(b c)(c a)(a + b + c): (Võ Quốc Bá Cẩn) Lời giải. Nếu a  b  c  0; thì P  0: Nếu c  b  a  0; thì P = (c b)(b a)(c a)(a + b + c) = (c  b)(b  a)(c 2 + bc  a 2  ab)  b(c  b)(c 2 + bc) = (c 2  bc)(b 2 + bc)  1 4  (b 2 + c 2 ) 2 = 1 4 239 Mặt khác, cho a = 0; b = sin  8 ; c = cos  8 ; ta có P = 1 4 : Vậy nên max P = 1 4 : Bài toán 2.34 Cho các số dương a; b; c; d: Chứng minh rằng b(a + c) c(a + b) + c(b + d) d(b + c) + d(c + a) a(c + d) + a(d + b) b(d + a)  4: (Võ Quốc Bá Cẩn) Lời giải. Viết lại bất đẳng thức như sau (a + c)  b c(a + b) + d a(c + d)  + (b + d)  c d(b + c) + a b(d + a)   4 , (abc + abd + acd + bcd)  a + c ac(a + b)(c + d) + b + d bd(b + c)(d + a)   4 ,  1 a + 1 b + 1 c + 1 d  1 a + 1 c  1 a + 1 b  1 c + 1 d  + 1 b + 1 d  1 b + 1 c  1 d + 1 a  !  4 Sử dụng bất đẳng thức AM-GM, ta có 1 a + 1 c  1 a + 1 b  1 c + 1 d  + 1 b + 1 d  1 b + 1 c  1 d + 1 a   4  1 a + 1 c   1 a + 1 b + 1 c + 1 d  2 + 4  1 b + 1 d   1 a + 1 b + 1 c + 1 d  2 = 4 1 a + 1 b + 1 c + 1 d : Bất đẳng thức được chứng minh. Đẳng thức xảy ra khi và chỉ khi a = c và b = d: Bài toán 2.35 Cho các số dương a; b; c: Chứng minh rằng a 2 + bc a 2 + (b + c) 2 + b 2 + ca b 2 + (c + a) 2 + c 2 + ab c 2 + (a + b) 2  18 5  a 2 + b 2 + c 2 (a + b + c) 2 : (Phạm Hữu Đức) Lời giải. Bất đẳng thức đã cho tương đương với X cy c (b + c) 2  bc a 2 + (b + c) 2 + 18 5  a 2 + b 2 + c 2 (a + b + c) 2  3 240 CHƯƠNG 2. SÁNG TẠO BẤT ĐẲNG THỨC Do (b + c) 2  4bc, nên ta chỉ cần chứng minh được X cy c (b + c) 2 4[a 2 + (b + c) 2 ] + 6 5  a 2 + b 2 + c 2 (a + b + c) 2  1 Sử dụng bất đẳng thức Cauchy Schwarz, ta có X cy c (b + c) 2 4[a 2 + (b + c) 2 ]  (a + b + c) 2 P cy c [a 2 + (b + c) 2 ] = (a + b + c) 2 2(a 2 + b 2 + c 2 ) + (a + b + c) 2 Chuẩn hóa cho a + b + c = 1. Đặt x = a 2 + b 2 + c 2 ) 3x  1, ta phải chứng minh 1 2x + 1 + 6x 5  1 , x(3x 1)  0: Đẳng thức xảy ra khi và chỉ khi a = b = c: Bài toán 2.36 Cho các số dương a; b; c; d: Chứng minh rằng (a + b)(a + c)(a + d)(b + c)(b + d)(c + d)  4(abc + bcd + cda + dab) 2 : (Võ Quốc Bá Cẩn) Lời giải. Sử dụng bất đẳng thức Cauchy Schwarz, ta có (abc + bcd + cda + dab) 2  (ac + bc + ad + bd)(ab 2 c + bcd 2 + ac 2 d + a 2 bd) = (a + b)(c + d)(ab 2 c + bcd 2 + ac 2 d + a 2 bd) (abc + bcd + cda + dab) 2  (bc + bd + ac + ad)(a 2 bc + bc 2 d + acd 2 + ab 2 d) = (a + b)(c + d)(a 2 bc + bc 2 d + acd 2 + ab 2 d) Công tương ứng vế với vế 2 bất đẳng thức trên, ta được 2(abc + bcd + cda + dab) 2  (a + b)(c + d)(ab 2 c + bcd 2 + ac 2 d + a 2 bd + a 2 bc + bc 2 d + acd 2 + ab 2 d) = (a + b) 2 (c + d) 2 (ab + cd) Tương tự, ta cũng có 2(abc + bcd + cda + dab) 2  (a + c) 2 (b + d) 2 (ac + bd) 2(abc + bcd + cda + dab) 2  (a + d) 2 (b + c) 2 (ad + bc) 241 Nhân tương ứng vế với vế, ta được 8(abc + bcd + cda + dab) 6  (ab + cd)(ac + bd)(ad + bc) Y sy m (a + b) 2 Mặt khác, theo bất đẳng thức AM-GM thì 4(ab + cd)(ac + bd)  (ab + cd + ac + bd) 2 = (a + d) 2 (b + c) 2 Tương tự, 4(ab + cd)(ad + bc)  (a + c) 2 (b + d) 2 4(ac + bd)(ad + bc)  (a + b) 2 (c + d) 2 Do đó 64(ab + cd) 2 (ac + bd) 2 (ad + bc) 2  Y sy m (a + b) 2 , (ab + cd)(ac + bd)(ad + bc)  1 8 Y sy m (a + b) Kết hợp với bất đẳng thức 8(abc+bcd+cda+dab) 6  (ab+cd)(ac+bd)(ad+bc) Q sy m (a+ b) 2 , ta suy ra được 8(abc + bcd + cda + dab) 6  1 8 " Y sy m (a + b) #" Y sy m (a + b) 2 # = 1 8 Y sy m (a + b) 3 : Từ đây, ta suy ra đpcm. Đẳng thức xảy ra khi và chỉ khi a = b = c = d: Bài toán 2.37 Cho các số dương a; b; c: Chứng minh rằng 3 + a b + b c + c a  2 s (a + b + c)  1 a + 1 b + 1 c  : (Phạm Hữu Đức) Lời giải. Đặt x 3 = a; y 3 = b; z 3 = c. Sử dụng bất đẳng thức Schur bậc 3, ta có 3 + a b + b c + c a = 3 x y  y z  z x + x 3 y 3 + y 3 z 3 + z 3 x 3  X cy c x 2 y 2  y z + z x  = X cy c x 2 yz + X cy c xz y 2 242 CHƯƠNG 2. SÁNG TẠO BẤT ĐẲNG THỨC Mặt khác, theo bất đẳng thức AM-GM thì X cy c x 2 yz + X cy c xz y 2 = x 3 + y 3 + z 3 xyz + (xz) 3 + (yx) 3 + (zy) 3 (xyz) 2  2 s (x 3 + y 3 + z 3 )(x 3 z 3 + y 3 x 3 + z 3 y 3 ) x 3 y 3 z 3 = 2 s (x 3 + y 3 + z 3 )  1 x 3 + 1 y 3 + 1 z 3  = 2 s (a + b + c)  1 a + 1 b + 1 c  : Đẳng thức xảy ra khi và chỉ khi a = b = c: Bài toán 2.38 Cho các số dương a; b; c. Chứng minh rằng abc + 2(a 2 + b 2 + c 2 ) + 8  5(a + b + c): (Trần Nam Dũng) Lời giải. Sử dụng bất đẳng thức AM-GM và bất đẳng thức Schur bậc 3, ta có 6abc + 12(a 2 + b 2 + c 2 ) + 48  30(a + b + c) = 3(2abc + 1) + 12(a 2 + b 2 + c 2 ) + 45  5  2  (a + b + c)  3  9 3 p a 2 b 2 c 2 + 12(a 2 + b 2 + c 2 ) + 45  5[(a + b + c) 2 + 9] = 9abc 3 p abc + 7(a 2 + b 2 + c 2 )  10(ab + bc + ca)  27abc a + b + c + 7(a 2 + b 2 + c 2 )  10(ab + bc + ca)  3[4(ab + bc + ca)  (a + b + c) 2 ] + 7(a 2 + b 2 + c 2 )  10(ab + bc + ca) = 4(a 2 + b 2 + c 2  ab  bc  ca)  0: Đẳng thức xảy ra khi và chỉ khi a = b = c = 1: Bài toán 2.39 Cho các số không âm a; b; c thỏa mãn ab + bc + ca + 6abc = 9: Tìm hằng số k nhỏ nhất sao cho bất đẳng thức sau đúng a + b + c + kabc  k + 3: (Võ Quốc Bá Cẩn) [...]... a 3 ; bất đẳng thức là hiển 1 ; ta có thể dễ dàng kiểm tra được 3 p p bc a b2 + c2 a2 = a 9 2a2 Do đó, ta phải chứng minh a2 p 9 2a2 + 1 3a 246 Nếu 1 CHƯƠNG 2 SÁNG TẠO BẤT ĐẲNG THỨC a 1 3; , f (a) = 2a6 6a + 1 0 ta có 2f (a) = a2 (a 1) 4a(a p Nếu 3 a 1; thì f (a) 9a4 + 9a2 2a6 1)2 + 12a2 9a4 + 9a2 18a + 7 + (1 5 = (a2 1)(2a4 3a)(7a2 7a2 + 2) 6a + 3) 3 0 3 . bc + ca; r = abc thì ta có q + 6r = 9. Sử dụng bất đẳng thức AM-GM, ta có p 2  3q  9. Bất đẳng thức trở thành p + 3r  6 , 2p q  3 Nếu p  6, bất đẳng thức là hiển nhiên. Nếu 6  p  3 và. 7q 3  9qr = 1  22 3 q + 49 3 q 2  7q 3  9qr Từ bất đẳng thức (a  b) 2 (b  c) 2 (c  a) 2  0, ta suy ra được r  1 27 h 9q  2 + 2(1  3q) p 1  3q i 236 CHƯƠNG 2. SÁNG TẠO BẤT ĐẲNG THỨC Do. P cyc a ! 2  1 3 thì bất đẳng thức trở thành 3(1  2t) + 2 2(1  2t) + t  3 , 2(1  3t) 2 2  3t  0: Bất đẳng thức cuối hiển nhiên đúng nên ta có đpcm. Đẳng thức xảy ra khi và chỉ khi x = y = z: Bài toán 2.30

Ngày đăng: 30/07/2014, 14:21

Từ khóa liên quan

Tài liệu cùng người dùng

  • Đang cập nhật ...

Tài liệu liên quan